Particle with spin 1 in a non perturtuvative interaction

So, you can't have|s_z = +\hbar> = a |u1> + b |u2> + c |u3> because the right hand side is not a column vector. It's just a sum of column vectors. So, you can't have|s_z = +\hbar> = a |u1> + b |u2> + c |u3> because the right hand side is not a column vector. It's just a sum of column vectors. So, you can't have
  • #1
WarDieS
23
0
Hi, i have been trying to figure how to solve this problem, i just don't know how to proceed

Homework Statement


A particle with spin s=1 has due to rotational invariance all three states [itex]|s_z\rangle=1[/itex], [itex]|s_z\rangle=0[/itex] and [itex]|s_z\rangle=-1[/itex] degenated with energy [itex]E_0>0[/itex]. Then its introduced an interaction non perturbative V

[itex]V=-\Delta\left(\begin{array}{ccc}
1 & 1 & 1\\
1 & 1 & 1\\
1 & 1 & 1
\end{array}\right)
[/itex]

a) Obtain stationary states and energies of the system
b) If in [itex]t=0[/itex], the particle has its third component with spin [itex]+\hbar[/itex] which is the probability that after some time its [itex]-\hbar[/itex]

Homework Equations


The Attempt at a Solution


a)
So what i tried is this, since all three states have the same energy and those are eigenstates of the hamiltonian, the hamiltonian must have the form

[itex]H=E_0\left(\begin{array}{ccc}
1 & 0 & 0\\
0 & 1 & 0\\
0 & 0 & 1
\end{array}\right)
[/itex]

So if we add H'=H+V and diagonalize we get

[itex]\left|\begin{array}{ccc}
E_0-\Delta-\lambda & -\Delta & -\Delta\\
-\Delta & E_0-\Delta-\lambda & -\Delta\\
-\Delta & -\Delta & E_0-\Delta-\lambda
\end{array}\right|
[/itex]
So the characteristic equation is
[itex](E_0-\lambda)(E_0-3\Delta-\lambda)=0[/itex]
So we get the energies and the states
[itex](E_0,{-1,0,1})[/itex]
[itex](E_0,{-1,1,0})[/itex]
[itex](E_0-3\Delta,{1,1,1})[/itex]

I just want to know if this is the correct move or not

b)
i don't know how to approach this, but i guess all i have to do is add the evolution term on each eigenstate [itex]e^{-i E_0 t/\hbar}[/itex] and do [itex]v^\dagger v[/itex]

Thanks for your help :)
 
Last edited:
Physics news on Phys.org
  • #2
WarDieS said:
So we get the energies and the states
[itex](E_0,{-1,0,1})[/itex]
[itex](E_0,{-1,1,0})[/itex]
[itex](E_0-3\Delta,{1,1,1})[/itex]

I just want to know if this is the correct move or not

Yes, that looks good!

... but i guess all i have to do is add the evol ution term on each eigenstate [itex]e^{-i E_0 t/\hbar}[/itex] and do [itex]v^\dagger v[/itex]

I think you might have the right idea. Can you "expand" the ##|s_z=+\hbar \rangle## state in terms of your eigenstates of the full Hamiltonian?
 
Last edited:
  • #3
I am lost on this, what i think may work is this
I have to solve the equation

[itex]i\hbar\frac{\partial}{\partial x} \chi = H \chi [/itex]

where
[itex]
\chi=\left(\begin{array}{ccc}
\alpha(t) \\
\beta(t) \\
\gamma(t)
\end{array}\right)
[/itex]

The problem with this is that this gives a really complicated result with the system
[itex]
i \hbar \dot{\alpha}(t)=(E_0-\Delta)\alpha(t)-\Delta(\beta(t)+\gamma(t))
[/itex]
[itex]
i \hbar \dot{\beta}(t)=(E_0-\Delta)\beta(t)-\Delta(\alpha(t)+\gamma(t))
[/itex]
[itex]
i \hbar \dot{\gamma}(t)=(E_0-\Delta)\gamma(t)-\Delta(\beta(t)+\alpha(t))
[/itex]

And this seems wrong to me

Thanks for help :)
 
  • #4
At time t = 0, you know the state is ##|\psi(0)\rangle = |s_z = +\hbar\rangle##. How would you represent this state as a column vector?

Can you expand this column vector in terms of the eigenvetors
[itex]|u1\rangle =
\left(\begin{array}{ccc}
-1 \\
0 \\
1
\end{array}\right)
[/itex] [itex],|u2\rangle =
\left(\begin{array}{ccc}
-1 \\
1 \\
0
\end{array}\right)
[/itex] [itex],|u3\rangle =
\left(\begin{array}{ccc}
1 \\
1 \\
1
\end{array}\right)
[/itex] ?
 
  • #5
So if if we normalize the vector we have

[itex]
|u_1\rangle = \frac{1}{\sqrt{2}} \left(\begin{array}{ccc} -1 \\ 0 \\ 1\end{array}\right)
[/itex]

[itex]
|u_2\rangle = \frac{1}{\sqrt{2}} \left(\begin{array}{ccc} -1 \\ 1 \\ 0\end{array}\right)
[/itex]

[itex]
|u_3\rangle = \frac{1}{\sqrt{3}} \left(\begin{array}{ccc} 1 \\ 1 \\ 1\end{array}\right)
[/itex]
so [itex]a|u_1\rangle+b|u_2\rangle+c|u_3\rangle=\left(\begin{array}{ccc} 1 \\ 0 \\ 0\end{array}\right)[/itex]

gives us a system, when we solve this we have the vector

[itex] |s_z{_h}\rangle=\left(\begin{array}{ccc} \frac{-\sqrt{2}}{3} \\ \frac{-\sqrt{2}}{3} \\ \frac{1}{3}\end{array}\right)
[/itex]

Now its turn for the time evolution
[itex]|s_z=\hbar\rangle=\left(\begin{array}{ccc} \frac{-\sqrt{2}}{3}\exp{\frac{-i E_1 t}{\hbar}} \\ \frac{-\sqrt{2}}{3}\exp{\frac{-i E_2 t}{\hbar}} \\ \frac{1}{3}\exp{\frac{-i E_3 t}{\hbar}}\end{array}\right)
[/itex]

Since i am being asked for [itex]-\hbar[/itex], i think i need to do this with that vector

[itex]|s_z=-\hbar\rangle=\left(\begin{array}{ccc} \frac{2\sqrt{2}}{3}\exp{\frac{-i E_1 t}{\hbar}} \\ \frac{-\sqrt{2}}{3}\exp{\frac{-i E_2 t}{\hbar}} \\ \frac{1}{3}\exp{\frac{-i E_3 t}{\hbar}}\end{array}\right)
[/itex]

Now ot obtain the evolution with time i have to get the expectation value i assume

[itex] \langle s_z\rangle=\langle s_z=-\hbar|S_z|s_z=-\hbar\rangle [/itex]

Is this good?

Thanks again for your help :D
 
  • #6
Your expressions for a, b, and c look good.

You didn't quite evolve in time correctly. It's only the eigenstates of H (i.e., |u1>|,u2>,|u3>) that have the simple time evolution of factors of exp(-iEt/[itex]\hbar[/itex]), for appropriate values of E.

So, you found ψ(0) = a |u1> + b |u2> + c |u3>.

To find ψ(t), let each of the |u> states evolve in time with it's own exponential factor. Then combine into one column vector.
 
  • #7
Your expressions for a, b, and c look good. But, you didn't quite evolve in time correctly. It's only the eigenstates of H (i.e., |u1>|,u2>,|u3>) that have the simple time evolution of factors of exp(-iEt/[itex]\hbar[/itex]), for appropriate values of E.

So, you found ψ(0) = a |u1> + b |u2> + c |u3>.

To find ψ(t), let each of the |u> states evolve in time with it's own exponential factor. Then combine into one column vector.

[EDIT: For example, at time t = 0 we have a|u1> = a ##\left(\begin{array}{ccc}
-1\\
0\\
1 \end{array}\right)##. To evolve this in time just multiply by a factor of ##e^{-iE_0 t/\hbar}##.]

EDIT 2: Also, as I understand it, you are not looking for ##\langle s_z \rangle##. Rather your a looking for the probability that at time t the system will be in the state ##|s_z = -\hbar \rangle = \left(\begin{array}{ccc}
0\\
0\\
1 \end{array}\right)##
 
Last edited:
  • #8
I have been assuming that we always write our column vectors with respect to the basis in which you wrote your matrices for ##H_o## and ##V##. This is the basis where

##\left(\begin{array}{ccc} 1\\ 0\\ 0 \end{array}\right) = |s_{z=\hbar}\rangle## = state where z-component of spin is ##+\hbar##

##\left(\begin{array}{ccc} 0\\ 1\\ 0 \end{array}\right) = |s_{z=0}\rangle## = state where z-component of spin is 0

##\left(\begin{array}{ccc} 0\\ 0\\ 1 \end{array}\right) = |s_{z=-\hbar}\rangle## = state where z-component of spin is ##-\hbar##

It appears to me that you might be trying to switch to writing column vectors such that

##\left(\begin{array}{ccc} 1\\ 0\\ 0 \end{array}\right) = |u_1\rangle## , ##\left(\begin{array}{ccc} 0\\ 1\\ 0 \end{array}\right) = |u_2\rangle##, ##\left(\begin{array}{ccc} 0\\ 0\\ 1 \end{array}\right) = |u_3\rangle##

You can't do that since ##|u1\rangle## and ##|u2\rangle## are not orthogonal.
 
  • #9
Ok, after some thinking i did this
I get this vector after diagonalizing the full hamiltonian and now i make sure they are orthogonal to each other

##|u_1\rangle=\frac{1}{\sqrt{2}}\left(\begin{array}{ccc} 1\\ -1\\ 0 \end{array}\right) = |s_{z=\hbar}\rangle##

##|u_1\rangle=\frac{1}{\sqrt{6}}\left(\begin{array}{ccc} 1\\ 1\\ -2 \end{array}\right) = |s_{z=\hbar}\rangle##

##|u_1\rangle=\frac{1}{\sqrt{3}}\left(\begin{array}{ccc} 1\\ 1\\ 1 \end{array}\right) = |s_{z=\hbar}\rangle##

Expanding the initial function and adding the time evolution terms

## |i\rangle = \sum_i \psi_i \langle \psi_i | i\rangle ##

##|i\rangle=\frac{1}{\sqrt{2}}|u_1\rangle e^{\frac{-i E_0 t}{\hbar}}+\frac{1}{\sqrt{6}}|u_2\rangle e^{\frac{-i E_0 t}{\hbar}}+\frac{1}{\sqrt{3}}|u_3\rangle e^{\frac{-i (E_0-3\Delta) t}{\hbar}} ##


Now we have the initial state expanded using the full hamiltonian eigenstates, we must find the amplitude with the vector

## |s_z=-\hbar\rangle=\left(\begin{array}{ccc} 0\\ 0\\ 1 \end{array}\right) ##

that is

## \langle s_z=-\hbar|i\rangle=\left(\frac{-1}{3}+\frac{e^{i 3 \Delta t/\hbar}}{3}\right)e^{-i E_0 t/\hbar} = c_i ##

now its time to get the probability
##p(t)=|c_i|^2=\frac{1}{9}\left(2-2\cos(3\Delta t/\hbar\right) ##

Wich i really hope its the right answer :/ , atleast ## p(0)=0 ## which is a necessary condition.
 
  • #10
Yes. Very nicely done!
 
  • Like
Likes 1 person
  • #11
i am really grateful, thanks for your patience with me :D
 

Related to Particle with spin 1 in a non perturtuvative interaction

1. What is a particle with spin 1 in a non perturtuvative interaction?

A particle with spin 1 in a non perturtuvative interaction refers to a subatomic particle that has an intrinsic angular momentum of 1, which determines its behavior in a non perturbative interaction. This means that its spin plays a crucial role in how it interacts with other particles and is not easily described by perturbation theory.

2. How does a particle with spin 1 differ from a particle with spin 0 or 1/2?

A particle with spin 1 differs from a particle with spin 0 or 1/2 in terms of its intrinsic angular momentum. Spin 0 particles have no angular momentum, while spin 1/2 particles have half of the intrinsic angular momentum of a spin 1 particle. This affects how they interact with other particles and their properties, such as mass and charge.

3. What are some examples of particles with spin 1?

Some examples of particles with spin 1 include the photon, W and Z bosons, and the rho meson. These particles are all fundamental particles, meaning they cannot be broken down into smaller components, and play important roles in the fundamental forces of nature.

4. How is the spin of a particle with spin 1 measured?

The spin of a particle with spin 1 can be measured using various experimental techniques, such as scattering experiments or observing the particle's interactions with other particles. It can also be calculated using theoretical models and equations, based on the particle's properties and behavior.

5. What is the significance of a particle with spin 1 in non perturbative interactions?

A particle with spin 1 in non perturbative interactions has a significant role in understanding the behavior of subatomic particles and the fundamental forces of nature. Its spin determines how it interacts with other particles and is crucial in describing the strong and weak nuclear forces, and the electromagnetic force.

Similar threads

  • Advanced Physics Homework Help
Replies
16
Views
2K
  • Advanced Physics Homework Help
Replies
4
Views
948
  • Advanced Physics Homework Help
Replies
9
Views
950
  • Advanced Physics Homework Help
Replies
9
Views
2K
  • Advanced Physics Homework Help
Replies
19
Views
2K
  • Advanced Physics Homework Help
Replies
5
Views
890
Replies
3
Views
891
  • Advanced Physics Homework Help
Replies
4
Views
826
  • Advanced Physics Homework Help
Replies
3
Views
450
  • Quantum Interpretations and Foundations
3
Replies
79
Views
4K
Back
Top